Infinite Sum of 1/n^2 Variant

An advanced problem one of my friends gave me which I think would be interesting to discuss: Compute n=11n2+1. \text {Compute } \displaystyle\sum_{n=1}^{\infty} \dfrac {1}{n^2+1}.

#MathProblem #Math

Note by Ahaan Rungta
7 years, 10 months ago

No vote yet
19 votes

  Easy Math Editor

This discussion board is a place to discuss our Daily Challenges and the math and science related to those challenges. Explanations are more than just a solution — they should explain the steps and thinking strategies that you used to obtain the solution. Comments should further the discussion of math and science.

When posting on Brilliant:

  • Use the emojis to react to an explanation, whether you're congratulating a job well done , or just really confused .
  • Ask specific questions about the challenge or the steps in somebody's explanation. Well-posed questions can add a lot to the discussion, but posting "I don't understand!" doesn't help anyone.
  • Try to contribute something new to the discussion, whether it is an extension, generalization or other idea related to the challenge.
  • Stay on topic — we're all here to learn more about math and science, not to hear about your favorite get-rich-quick scheme or current world events.

MarkdownAppears as
*italics* or _italics_ italics
**bold** or __bold__ bold

- bulleted
- list

  • bulleted
  • list

1. numbered
2. list

  1. numbered
  2. list
Note: you must add a full line of space before and after lists for them to show up correctly
paragraph 1

paragraph 2

paragraph 1

paragraph 2

[example link](https://brilliant.org)example link
> This is a quote
This is a quote
    # I indented these lines
    # 4 spaces, and now they show
    # up as a code block.

    print "hello world"
# I indented these lines
# 4 spaces, and now they show
# up as a code block.

print "hello world"
MathAppears as
Remember to wrap math in \( ... \) or \[ ... \] to ensure proper formatting.
2 \times 3 2×3 2 \times 3
2^{34} 234 2^{34}
a_{i-1} ai1 a_{i-1}
\frac{2}{3} 23 \frac{2}{3}
\sqrt{2} 2 \sqrt{2}
\sum_{i=1}^3 i=13 \sum_{i=1}^3
\sin \theta sinθ \sin \theta
\boxed{123} 123 \boxed{123}

Comments

Let f(x)=coshxf(x) = \cosh x, and let's try to find the Fourier Series of f(x)f(x) on [π,π][-\pi,\pi].

cn=12πππeinxcoshxdxc_n = \dfrac{1}{2\pi}\displaystyle\int_{-\pi}^{\pi}e^{-inx}\cosh x\,dx =14πππeinx(ex+ex)dx= \dfrac{1}{4\pi}\displaystyle\int_{-\pi}^{\pi}e^{-inx}(e^x+e^{-x})\,dx =14πππ(e(1in)x+e(1in)x)dx= \dfrac{1}{4\pi}\displaystyle\int_{-\pi}^{\pi}(e^{(1-in)x}+e^{(-1-in)x})\,dx

=14π[e(1in)x1in+e(1in)x1in]ππ= \dfrac{1}{4\pi}\left[\dfrac{e^{(1-in)x}}{1-in} + \dfrac{e^{(-1-in)x}}{-1-in}\right]_{-\pi}^{\pi} =14π[e(1in)πe(1in)π1in+e(1in)πe(1in)π1in]= \dfrac{1}{4\pi}\left[\dfrac{e^{(1-in)\pi} - e^{-(1-in)\pi}}{1-in} + \dfrac{e^{(-1-in)\pi} - e^{-(-1-in)\pi}}{-1-in}\right]

=(1)n4π[eπeπ1in+eπeπ1in]= \dfrac{(-1)^n}{4\pi}\left[\dfrac{e^{\pi} - e^{-\pi}}{1-in} + \dfrac{e^{-\pi} - e^{\pi}}{-1-in}\right] =(1)nsinhπ2π[11in11+in]= \dfrac{(-1)^n\sinh \pi}{2\pi}\left[\dfrac{1}{1-in} - \dfrac{1}{1+in}\right] =(1)nsinhππ(n2+1)= \dfrac{(-1)^n\sinh \pi}{\pi(n^2+1)}.

Therefore, f(x)=coshx=n=(1)nsinhππ(n2+1)einxf(x) = \cosh x = \displaystyle\sum_{n = -\infty}^{\infty}\dfrac{(-1)^n\sinh \pi}{\pi(n^2+1)}e^{inx}.

Thus, f(π)=coshπ=n=sinhππ(n2+1)f(\pi) = \cosh \pi = \displaystyle\sum_{n = -\infty}^{\infty}\dfrac{\sinh \pi}{\pi(n^2+1)}, and so, n=1n2+1=πcoshπsinhπ\displaystyle\sum_{n = -\infty}^{\infty}\dfrac{1}{n^2+1} = \dfrac{\pi \cosh \pi}{\sinh \pi}.

Subtracting the n=0n = 0 term and dividing by 22 gives: n=11n2+1=πcoshπ2sinhπ12=π2cothπ12\displaystyle\sum_{n = 1}^{\infty}\dfrac{1}{n^2+1} = \dfrac{\pi \cosh \pi}{2\sinh \pi} - \dfrac{1}{2} = \dfrac{\pi}{2}\coth \pi - \dfrac{1}{2}.

EDIT: This is essentially what C L. did, except with complex Fourier coefficients instead.

Jimmy Kariznov - 7 years, 10 months ago

Log in to reply

We both posted the exact same solution at the same time. :D

C Lim - 7 years, 10 months ago

Bob's right: the answer reminded me of some Fourier transform computations I did some years back.

Let f(x)=cosh(x)f(x) = \cosh(x) on the interval (π,+π] (-\pi, +\pi] and repeated periodically through the real line via f(x+2π)=f(x)f(x+2\pi) = f(x). It's continuous and nice enough that its Fourier series converges pointwise.

Convergence of Fourier Transform

Specifically, at the point x=πx = \pi, the one-sided derivatives exist:

limxπf(x)coshπxπ=sinhπ,limxπ+f(x)coshπxπ=sinh(π)=sinhπ \lim_{x \to \pi^-} \frac{f(x) - \cosh\pi} {x - \pi} = \sinh\pi, \quad \lim_{x\to \pi^+} \frac{f(x) - \cosh\pi} {x - \pi} = \sinh(-\pi) = -\sinh \pi

where the second equality follows from f(x)=cosh(x2π) f(x) = \cosh(x - 2\pi) when x is slightly more than 2π2 \pi. Furthermore, the one-sided derivatives of f(x)f(x) have only discrete discontinuities. One can then apply Dini criterion to show that the Fourier series converges.

Computing the Fourier Transform

Omitting the details, the Fourier coefficients of f(x) f(x) are given by:

an=1πππf(x)cosnxdx=1πππcoshxcosnxdx=2(1)nsinhππ,bn=1πππf(x)sinnxdx=1πππcoshxsinnxdx=0. \begin{aligned} a_n &= \frac 1 \pi \int_{-\pi}^{\pi} f(x)\cos nx dx = \frac 1 \pi \int_{-\pi}^\pi \cosh x \cos nx dx = 2(-1)^n \frac{\sinh \pi}{\pi},\\ b_n &= \frac 1 \pi \int_{-\pi}^{\pi} f(x)\sin nx dx = \frac 1 \pi \int_{-\pi}^\pi \cosh x \sin nx dx = 0.\end{aligned}

Thus, the Fourier expansion gives:

f(x)=12a0+n=1(ancosnx+bnsinnx).=sinhππ(1+2n=1(1)ncosnxn2+1) \begin{aligned} f(x) &= \frac 1 2 a_0 + \sum_{n=1}^\infty (a_n \cos nx + b_n \sin nx). \\ &= \frac{\sinh \pi}\pi \left( 1 + 2\sum_{n=1}^\infty (-1)^n \frac {\cos nx} {n^2 + 1}\right) \end{aligned}

Substituting x=πx = \pi then gives:

coshπ=sinhππ(1+2n=11n2+1) \cosh \pi = \frac{\sinh \pi}\pi \left( 1 + 2\sum_{n=1}^\infty \frac 1 {n^2 + 1}\right)

which gives Bob's answer.

C Lim - 7 years, 10 months ago

Here's a shorter way.

Note that cot(x)=1x+k=12xx2k2π2 \cot( x)=\frac{1}{x}+ \sum_{k=1}^{\infty} \frac{2x}{x^2-k^2 \pi^2} (Proof: use infinite product for sine, take natural logs, differentiate, rearrange). Thus,

1π2k=12xx2π2k2=cot(x)1x\large \frac{1}{\pi^2} \sum_{k=1}^{\infty} \frac{2x}{\frac{x^2}{\pi^2}-k^2 }=\cot( x)-\frac{1}{x}

k=1xk2x2π2=π22(cot(x)1x)\large \sum_{k=1}^{\infty} \frac{x}{k^2-\frac{x^2}{\pi^2} }=\frac{-\pi^2}{2} \left(\cot( x)-\frac{1}{x}\right)

Let x=iπx=i\pi,

k=1iπk2+1=π22(cot(iπ)1iπ)\large \sum_{k=1}^{\infty} \frac{i \pi}{k^2+1 }=\frac{-\pi^2}{2} \left(\cot( i \pi)-\frac{1}{i \pi}\right)

k=11k2+1=iπ2(cot(iπ)1iπ)\large \sum_{k=1}^{\infty} \frac{1}{k^2+1 }=\frac{i\pi}{2} \left(\cot( i \pi)-\frac{1}{i \pi}\right)

k=11k2+1=iπ2cos(iπ)sin(iπ)12\large \sum_{k=1}^{\infty} \frac{1}{k^2+1 }=\frac{i\pi}{2} \frac{\cos(i \pi)}{\sin(i \pi)}-\frac{1}{2}

k=11k2+1=iπ2cosh(π)isinh(π)12\large \sum_{k=1}^{\infty} \frac{1}{k^2+1 }=\frac{i\pi}{2} \frac{\cosh( \pi)}{i\sinh( \pi)}-\frac{1}{2}

k=11k2+1=π2cosh(π)sinh(π)12\large \sum_{k=1}^{\infty} \frac{1}{k^2+1 }=\frac{\pi}{2} \frac{\cosh( \pi)}{\sinh( \pi)}-\frac{1}{2}

A L - 7 years, 10 months ago

Mathematica gave 12+π2cothπ\frac{-1}{2} + \frac{\pi}{2}\coth \pi. I know this isn't exactly the point, but sometimes the answer is the best place to start a question like this.

Bob Krueger - 7 years, 10 months ago

Is there a systematic way to approach these questions? I've seen Fourier Series solutions before, but they always struck me as most likely discovered by accident. For a simpler example, why would you immediately think to consider the Fourier Series of f(x)=xf(x) = |x| if you wanted to find ζ(2)\zeta(2)?

Eric Edwards - 7 years, 10 months ago

Log in to reply

The second answer here http://math.stackexchange.com/questions/8337/different-methods-to-compute-sum-limits-n-1-infty-frac1n2 makes the choice much more obvious (albeit it uses the function x2x^2, not x|x|, although of course these choices would yield identical results, just square at the end).

A L - 7 years, 10 months ago

Excellent work, everybody!

Ahaan Rungta - 7 years, 10 months ago
×

Problem Loading...

Note Loading...

Set Loading...